+0  
 
+1
3146
1
avatar+619 

Let $$f(x) = \frac{x^2}{x^2 - 1}.$$Find the largest integer $n$ so that $f(2) \cdot f(3) \cdot f(4) \cdots f(n-1) \cdot f(n) < 1.98.$

 Oct 5, 2017
 #1
avatar+773 
+5

n = 98

 Jan 25, 2019

2 Online Users

avatar